Difference between revisions of "2016 AMC 12B Problems/Problem 25"

(Solution 2)
(Solution 2)
Line 27: Line 27:
 
==Solution 2==
 
==Solution 2==
 
Since the product <math>a_1a_2\cdots a_k</math> is an integer, the sum of the logarithms <math>\log _2 a_k</math> must be an integer. Multiply all of these logarithms by <math>19</math>, so that the sum must be a multiple of <math>19</math>. We take these vales modulo <math>19</math> to save calculation time. Using the recursion <math>a_n=a_{n-1}a_{n-2}^2</math>:
 
Since the product <math>a_1a_2\cdots a_k</math> is an integer, the sum of the logarithms <math>\log _2 a_k</math> must be an integer. Multiply all of these logarithms by <math>19</math>, so that the sum must be a multiple of <math>19</math>. We take these vales modulo <math>19</math> to save calculation time. Using the recursion <math>a_n=a_{n-1}a_{n-2}^2</math>:
<cmath>a_0=0,a_1=1\dots\implies 0,1,1,3,5,11,2,5,9,0,18,18,16,14,10,17,14,10,0\dots</cmath>
+
<cmath>a_0=0,a_1=1\dots\implies 0,1,1,3,5,11,2,5,9,0,18,18,16,14,8,17,14,10,0\dots</cmath>
 
Notice that <math>a_k+a_{k+9}\equiv 0\text{ mod }19</math>. The cycle repeats every <math>9+9=18</math> terms. Since <math>a_0=0</math> and <math>a_{18}=0</math>, we only need the first <math>17</math> terms to sum up to a multiple of <math>19</math>: <math>\boxed{\textbf{(A) }17}</math>. (NOTE: This solution proves 17 is the upper bound, but since 17  is the lowest answer choice, it is correct. To rigorously prove it, you will have to add up the mods listed until you get <math>0\pmod{19}</math>.
 
Notice that <math>a_k+a_{k+9}\equiv 0\text{ mod }19</math>. The cycle repeats every <math>9+9=18</math> terms. Since <math>a_0=0</math> and <math>a_{18}=0</math>, we only need the first <math>17</math> terms to sum up to a multiple of <math>19</math>: <math>\boxed{\textbf{(A) }17}</math>. (NOTE: This solution proves 17 is the upper bound, but since 17  is the lowest answer choice, it is correct. To rigorously prove it, you will have to add up the mods listed until you get <math>0\pmod{19}</math>.
  

Revision as of 00:15, 20 July 2016

Problem

The sequence $(a_n)$ is defined recursively by $a_0=1$, $a_1=\sqrt[19]{2}$, and $a_n=a_{n-1}a_{n-2}^2$ for $n\geq 2$. What is the smallest positive integer $k$ such that the product $a_1a_2\cdots a_k$ is an integer?

$\textbf{(A)}\ 17\qquad\textbf{(B)}\ 18\qquad\textbf{(C)}\ 19\qquad\textbf{(D)}\ 20\qquad\textbf{(E)}\ 21$

Solution 1

Let $b_i=19\text{log}_2a_i$. Then $b_0=0, b_1=1,$ and $b_n=b_{n-1}+2b_{n-2}$ for all $n\geq 2$. The characteristic polynomial of this linear recurrence is $x^2-x-2=0$, which has roots $2$ and $-1$.

Therefore, $b_n=k_12^{n}+k_2(-1)^n$ for constants to be determined $k_1, k_2$. Using the fact that $b_0=0, b_1=1,$ we can solve a pair of linear equations for $k_1, k_2$:

$k_1+k_2=0$ $2k_1-k_2=1$.

Thus $k_1=\frac{1}{3}$, $k_2=-\frac{1}{3}$, and $b_n=\frac{2^n-(-1)^n}{3}$.

Now, $a_1a_2\cdots a_k=2^{\frac{(b_1+b_2+\cdots+b_k)}{19}}$, so we are looking for the least value of $k$ so that

$b_1+b_2+\cdots+b_k \equiv 0 \pmod{19}$.

Note that we can multiply all $b_i$ by three for convenience, as the $b_i$ are always integers, and it does not affect divisibility by $19$.

Now, for all even $k$ the sum (adjusted by a factor of three) is $2^1+2^2+\cdots+2^k=2^{k+1}-2$. The smallest $k$ for which this is a multiple of $19$ is $k=18$ by Fermat's Little Theorem, as it is seen with further testing that $2$ is a primitive root $\pmod{19}$.

Now, assume $k$ is odd. Then the sum (again adjusted by a factor of three) is $2^1+2^2+\cdots+2^k+1=2^{k+1}-1$. The smallest $k$ for which this is a multiple of $19$ is $k=17$, by the same reasons. Thus, the minimal value of $k$ is $\textbf{(A) } 17$.

Solution 2

Since the product $a_1a_2\cdots a_k$ is an integer, the sum of the logarithms $\log _2 a_k$ must be an integer. Multiply all of these logarithms by $19$, so that the sum must be a multiple of $19$. We take these vales modulo $19$ to save calculation time. Using the recursion $a_n=a_{n-1}a_{n-2}^2$: \[a_0=0,a_1=1\dots\implies 0,1,1,3,5,11,2,5,9,0,18,18,16,14,8,17,14,10,0\dots\] Notice that $a_k+a_{k+9}\equiv 0\text{ mod }19$. The cycle repeats every $9+9=18$ terms. Since $a_0=0$ and $a_{18}=0$, we only need the first $17$ terms to sum up to a multiple of $19$: $\boxed{\textbf{(A) }17}$. (NOTE: This solution proves 17 is the upper bound, but since 17 is the lowest answer choice, it is correct. To rigorously prove it, you will have to add up the mods listed until you get $0\pmod{19}$.


$a_1a_2a_3a_4a_5a_6a_7a_8a_9a_{10}a_{11}a_{12}a_{13}a_{14}a_{15}a_{16}a_{17}=2^{87381/19}=2^{4599}\approx 2.735\cdot 10^{1384}$

See Also

2016 AMC 12B (ProblemsAnswer KeyResources)
Preceded by
Problem 24
Followed by
Last Problem
1 2 3 4 5 6 7 8 9 10 11 12 13 14 15 16 17 18 19 20 21 22 23 24 25
All AMC 12 Problems and Solutions

The problems on this page are copyrighted by the Mathematical Association of America's American Mathematics Competitions. AMC logo.png